Finding the Fourrier Coefficient

  • Thread starter Thread starter p0ssumZ
  • Start date Start date
  • Tags Tags
    Coefficient
Click For Summary
To find the Fourier coefficients for the function m(t) = cos(200πt) + sin(50πt), it is essential to understand Fourier series, which express functions as sums of sine and cosine terms. The given function already consists of these terms, making the calculation of the coefficients straightforward. It is recommended to search online for resources, such as Wikipedia, to grasp the concept and approach the problem effectively. Engaging with the material is crucial, as relying solely on others for solutions is not productive. Understanding the fundamentals will facilitate the calculation of the power of m(t) and its minimum and maximum values.
p0ssumZ
Messages
1
Reaction score
0

Homework Statement



the equation given is this one:

m(t)= cos(200\pit) + sin(50\pit)

Homework Equations



I need to find the Fourier coefficients for m(t), the power of m(t) and the min and max of m(t)

Fourier.jpg

The Attempt at a Solution



I tried to search online to find ways of determining the Fourier coefficients but I am really stuck. No lectures were given on that and our lecturer wants us to work it out still.

Anybody sees the light here? :cry:

Homework Statement

 
Physics news on Phys.org
hai it is better if you read about Fourier series. Many materials are there in internet. The basic idea is to express any function in terms of summation of sine and cosine terms and their integer harmonics.
In your problem the function itself is expressed as sum of cosine and a sine term. So the calculation of Fourier co-efficients is pretty much straight forward.
 
In summary, type "fourier coeffcients" in google. The first link is probably to wikipedia. Read it and give an attempt at the problem because anything less is just plain laziness.
 
Question: A clock's minute hand has length 4 and its hour hand has length 3. What is the distance between the tips at the moment when it is increasing most rapidly?(Putnam Exam Question) Answer: Making assumption that both the hands moves at constant angular velocities, the answer is ## \sqrt{7} .## But don't you think this assumption is somewhat doubtful and wrong?

Similar threads

  • · Replies 1 ·
Replies
1
Views
1K
  • · Replies 3 ·
Replies
3
Views
2K
  • · Replies 6 ·
Replies
6
Views
2K
  • · Replies 1 ·
Replies
1
Views
2K
  • · Replies 1 ·
Replies
1
Views
2K
  • · Replies 15 ·
Replies
15
Views
3K
  • · Replies 12 ·
Replies
12
Views
2K
  • · Replies 1 ·
Replies
1
Views
2K
  • · Replies 14 ·
Replies
14
Views
3K
  • · Replies 5 ·
Replies
5
Views
2K